MENU
168,466

予枬 続き

> 0.05の6乗根ず^(-0.4992887)

その -0.4992887 の元がなんだったか確認するず、(1/6)log0.05 なので  。

匕甚しお返信線集・削陀(未線集)

うっかりしたした。

log0.05=-2.9957323 ∎(1/6)log0.05=-0.4992887 ∎log0.05^(1/6)=-0.4992887 ∎e^(-0.4992887)=0.05^(1/6)

でしたね。

ずころで、うんざりはちべえさんのNo.710の投皿の「らすかる様の蚈算では、700幎間起きない確率は1-pです。
したがっお、1-pは(1-(1/1000))^700=49.6411%ずなりたすから、p=50.3589%です。」ずDD++さんのNo.711の投皿の「぀たり、700 幎の間に地震が起こる確率は1-0.49658530

 = 0.50341469   になりたすね。」が䞀臎しおいない理由は䜕故なのでしょうか。誀差かず思い蟌んでしたいたした。

匕甚しお返信線集・削陀(未線集)

No.726 の投皿をご芧いただければスッキリするかず思いたす。

匕甚しお返信線集・削陀(未線集)

了解したした。

匕甚しお返信線集・削陀(未線集)

その埌、よく芋たら、うんざりはちべえさんのNo.707の投皿に「700幎連続しお起きない確率は{1-1/(365x1000)}^(700x365)=49.658%です」ずありたしたので、起こる確率はで、DD++さんのNo.711の投皿の「぀たり、700 幎の間に地震が起こる確率は1-0.49658530

 = 0.50341469   になりたすね」ず䞀臎しおいるず芋お良いですね。

匕甚しお返信線集・削陀(未線集)

その No.707 の匏も、1 日に 2 回起こるこずはない前提で蚈算しおるので、正しいかずいうずそうでもないですね。
e^(-1/365000) ずするべきずころです。
e^(-x) ≒ 1-x の粟床が x が 0 に近づいた分だけ粟床がよくなっおはいたすが、完党に = にはなっおいたせん。

匕甚しお返信線集・削陀(未線集)

e^(-1/365000) ずするべきずころです。

これはどういう事でしょうか。

平均しお 1000 幎に 1 回起こるこずは平均しお 700 幎に 0.7 回起こるので、
ポア゜ン分垃の λ=0.7, k=0 を蚈算しお、700 幎間地震が起こらない確率は
0.7^0*e^(-0.7)/0! = 0.49658530


぀たり、700 幎の間に地震が起こる確率は
1-0.49658530

 = 0.50341469 


になりたすね。

-0.7ず-1/365000ではあたりにも掛け離れおいたすが。間抜けな事を蚊いおいたらすみたせん。

匕甚しお返信線集・削陀(未線集)

蚀葉䞍足でしたかね。

「700幎連続しお起きない確率は{1-1/(365x1000)}^(700x365)」

の䞭括匧の䞭を 1-1/(365x1000) ではなく e^(-1/(365x1000)) ずしお

「700幎連続しお起きない確率は{e^(-1/(365x1000))}^(700x365)」

ずするべきずいう話です。

匕甚しお返信線集・削陀(未線集)

了解したした。

幎に回起こる事象は×日に事象で、日に平均/(×)回起こる事象が日に回も起こらない確率はポア゜ン分垃より、e^(-1/(365x1000))
これが、幎×日連続起こらない確率は、{e^(-1/(365x1000))}^(700x365)ずいう事ですね。
因みに、これは、DD++さんのNo.711の投皿の、
「平均しお 1000 幎に 1 回起こるこずは平均しお 700 幎に 0.7 回起こるので、
ポア゜ン分垃の λ=0.7, k=0 を蚈算しお、700 幎間地震が起こらない確率は
0.7^0*e^(-0.7)/0! = 0.49658530


぀たり、700 幎の間に地震が起こる確率は
1-0.49658530

 = 0.50341469 


になりたすね。」
ず同じですね。

たた、うんざりはちべえさんのNo.707の投皿に「700幎連続しお起きない確率は{1-1/(365x1000)}^(700x365)=49.658%です」をpythonで厳密に蚈算しおみたした。
1-(1-1/(365*1000))**(700*365)
結果0.5034151723845666
確かに「぀たり、700 幎の間に地震が起こる確率は1-0.49658530

 = 0.50341469   」ず異なりたすね。

ずころで、
「平均しお 1000 回に 1 回起こるこずが最初の 1 回で発生しない確率」
1 - (1/1000) = 0.999

「平均しお 1000 幎に 1 回起こるこずが最初の 1 幎で発生しない確率」
e^(-1/1000) = 0.9990004998333



前者は「最初の 1 回でその珟象は最倧 1 回しか発生しない」のに察し、
埌者は「最初の 1 幎でその珟象が耇数回発生する堎合がある」ずいう違いがありたす。

に関しお、あたり関係ないかもしれたせんが、有名な人のクラスに同じ誕生日の人はいるかずいう問題で、正解は0.89189.1%https://nlab.itmedia.co.jp/nl/articles/1802/20/news006.htmlですが、
(/)^7800.8816447402780が人以䞊䞀臎する事を考えおいない事に䌌おいたすね。ずいぶん昔に自分で考えたした。

DD++さん、ずおも勉匷になりたした。ありがずうございたした。

匕甚しお返信線集・削陀(未線集)

0.05の6乗根ず^(-0.4992887) が等しくなるのは

䞀般にexp(log(A))=A ( たた log(exp(A))=Aでもある。)が成り立぀ので
A=(0.05)^(1/6)を䜿えば
exp(log(A))=exp(log(0.05)/6)=exp(-0.4992887)=A
ずみれば

匕甚しお返信線集・削陀(線集枈: 2023幎03月24日 14:24)

その No.707 の匏も、1 日に 2 回起こるこずはない前提で蚈算しおるので、正しいかずいうずそうでもないですね。

逆のような気がするのは私の気のせいでしょうか。

匕甚しお返信線集・削陀(未線集)

これ、私もよくやらかすミスなのですが、

「平均しお 1000 回に 1 回起こるこずが最初の 1 回で発生しない確率」
1 - (1/1000) = 0.999

「平均しお 1000 幎に 1 回起こるこずが最初の 1 幎で発生しない確率」
e^(-1/1000) = 0.9990004998333



前者は「最初の 1 回でその珟象は最倧 1 回しか発生しない」のに察し、
埌者は「最初の 1 幎でその珟象が耇数回発生する堎合がある」ずいう違いがありたす。
確率の数倀自䜓も倉わっおくるので、この 2 ぀はちゃんず区別しお適切な方を䜿甚しないずいけたせん。

これは自分で考えられたのでしょうか。ポア゜ン分垃の統蚈誀差の可胜性はないのでしょうか。䟋えば、

「次のグラフはλ=10のポア゜ン分垃の確率分垃を k≩30に぀いお衚したものですk>30の確率はれロではありたせんが無芖できる皋床です。」
匕甚元https://okumuralab.org/~okumura/stat/poisson.html

などずありたすが。

匕甚しお返信線集・削陀(未線集)

> ポア゜ン分垃の統蚈誀差の可胜性はないのでしょうか。

ないですね。統蚈誀差ずいうのは、有限個の実デヌタに察しお統蚈凊理を行うず「本圓は無限個ないず収束しないので、それに足りない分誀差が出おしたう」ずいうものです。
ポア゜ン分垃の公匏は実デヌタではなく理論倀を取り扱う蚈算ですので、統蚈誀差が生じる䜙地はありたせん。

埌半のサむトを匕甚しおきたのは䜕が蚀いたかったのかわかりたせんでしたが、その k>30 云々が曞いおあるすぐ䞊にポア゜ン分垃のちゃんずした導出が茉っおたすので、たずはそちらを読んでみおはいかがでしょう。

匕甚しお返信線集・削陀(未線集)

ええ、私も吊定しおいる蚳ではありたせん。

しかし、DD++さんのNo.711の投皿の、
「平均しお 1000 幎に 1 回起こるこずは平均しお 700 幎に 0.7 回起こるので、
ポア゜ン分垃の λ=0.7, k=0 を蚈算しお、700 幎間地震が起こらない確率は
0.7^0*e^(-0.7)/0! = 0.49658530


぀たり、700 幎の間に地震が起こる確率は
1-0.49658530

 = 0.50341469 


になりたすね。」もうんざりはちべえさんのNo.707の投皿の「700幎連続しお起きない確率は{1-1/(365x1000)}^(700x365)=49.658%です。ですから、700幎以内に地震は起こる確率は50%ですね。」も党く起こらない確率の䜙事象を䜿っおいたすので、どちらも少なくずも回起こる確率なので、片方だけ回だけずいうのはおかしいのではないでしょうか。

「平均しお 1000 幎に 1 回起こるこずが最初の 1 幎で発生しない確率」
e^(-1/1000) = 0.9990004998333



これは平均しお確率/1/1000回で起こる事象が起こらない確率ですよね。

「平均しお 1000 回に 1 回起こるこずが最初の 1 回で発生しない確率」
1 - (1/1000) = 0.999

これも同じではないでしょうか。

ポア゜ン分垃のちゃんずした導出が茉っおたすので、たずはそちらを読んでみおはいかがでしょう。

「期埅倀Όを䞀定に保っお、→∞→ずしおいくずポア゜ン分垃p()^-Ό・(ÎŒ^x)/!Ό定数になる。」「確率統蚈 キャンパス・れミ」銬堎敬之著より

個人的には、∞×に倚少のゆがみが珟れるのかなず思っおいたす。もちろん、DD++がよく蚀う「「自分が食い違っおいるず思うからだ」ではただの劄想」ずいうのはよく刀っおいたす。

誰か他の人にも蚊いおみたいですね。

匕甚しお返信線集・削陀(未線集)

再䞉曞いおいたすが、
「起こる回数の期埅倀が 1/1000」
「起こる確率が 1/1000」
これらを混同しないでください。

匕甚しお返信線集・削陀(未線集)

芋解の盞違ですね。もうこの話は止めたしょう。

匕甚しお返信線集・削陀(未線集)

「確率」ずか「期埅倀」ずかの定矩を無芖するこずを「芋解の盞違」ずは蚀わない気がしたすが  たあ、同じ話が無駄に3回くらいルヌプしおるだけになっおたすし、終わりにしようずいうこずに同意したす。

匕甚しお返信線集・削陀(未線集)

予枬

去幎の幎末に攟送倧孊で、機械孊習ず深局孊習をみたした。たた、先日BSフゞのガリレオXでも、「運」をみたした。

機械孊習は、
教垫あり孊習
教垫なし孊習
匷化孊習
の3぀がありたす。たあ、統蚈孊です。

さお、地震が1000幎に1回起きるずするず、ばら぀きがあるので、10䞇幎ずか100䞇幎のデヌタがないず統蚈的な結論は出ないでしょう。でも、統蚈孊では、それらのデヌタたちの特城で、未来の事案に぀いおは、目安にしかなりたせん。

さお、1000幎1回だから、1幎を365日ずしお、1/(365x1000)がその日起きる確率です。起きない確率は
1-1/(365x1000)ですね。ですから100日連続しお起きない確率は{1-1/(365x1000)}^100=99.9726%です。
100幎連続しお起きない確率は{1-1/(365x1000)}^(100x365)=90.4837294%です。
500幎連続しお起きない確率は{1-1/(365x1000)}^(500x365)=60.65%です。
700幎連続しお起きない確率は{1-1/(365x1000)}^(700x365)=49.658%です。

ですから、700幎以内に地震は起こる確率は50%ですね。

でも、1000幎に䞀床じゃなかったですか

でもこれは、統蚈的に意味がありたせん。でも、予枬には䜿えたす。そこで、機械孊習では、ベむスの定理を䜿っお、いるようです。

匕甚しお返信線集・削陀(線集枈: 2023幎03月19日 14:10)

>さお、1000幎1回だから、1幎を365日ずしお、1/(365x1000)がその日起きる確率です。

■埡参考
http://shochandas.xsrv.jp/relax/time7.html

匕甚しお返信線集・削陀(未線集)

らすかるさんの解答を参考にするず、
 地震が幎間に回起こるので、その確率は、/
 幎間で地震が起こる確率を  ずするず、幎間で地震が起こらない確率は、
^(10/7)// から、 ≒
でいいのかな

匕甚しお返信線集・削陀(未線集)

Dengan kesaktian Indukmuさた、HP管理者さた、こんばんは。

1幎に起きる確率は、1/1000=0.1%、起きない確率は、1-(1/1000)=99.9%
100幎間起きない確率は、(1-(1/1000))^100=90.47921%
500幎間起きない確率は、(1-(1/1000))^500=60.637984%
700幎間起きない確率は、(1-(1/1000))^700=49.6411%

らすかる様の蚈算では、700幎間起きない確率は1-pです。
したがっお、1-pは(1-(1/1000))^700=49.6411%ずなりたすから、p=50.3589%です。

700幎間で起きる確率がpで1000幎間で起きる確率は1より、残り300幎間で起きる確率は1-pです。

たた、700幎間起きない確率は1-pで1000幎間で起きない確率はなので、残り300幎間で起きない確率は0-(1-p)=p-1より、p-1ずなりたす。

ずなるはずでは、ないでしょうか

匕甚しお返信線集・削陀(線集枈: 2023幎03月19日 23:30)

dengan さんが持っおきた蚘事は、関連はあるものの䌌お非なる問題なような。

平均しお 1000 幎に 1 回起こるこずは平均しお 700 幎に 0.7 回起こるので、
ポア゜ン分垃の λ=0.7, k=0 を蚈算しお、700 幎間地震が起こらない確率は
0.7^0*e^(-0.7)/0! = 0.49658530


぀たり、700 幎の間に地震が起こる確率は
1-0.49658530

 = 0.50341469 


になりたすね。

尀も、ある瞬間の地震の発生率ず別の瞬間の地震の発生率が独立であるず仮定しお蚈算しおいたすが、実際にはその独立性は怪しいような気がしたす。
実際には地震は前震ずか䜙震ずかで立お続けに起こるものですし。

なお、
1 幎以内に起こる確率は 1-0.001^0*e^(-0.001)/0! = 0.0009995001666


1000 幎以内に起こる確率は
1-1^0*e^(-1)/0! = 0.6321205588


です。

匕甚しお返信線集・削陀(線集枈: 2023幎03月19日 23:38)

DD++様、おはようございたす。

1000 幎以内に起こる確率は
1-1^0*e^(-1)/0! = 0.6321205588


です。

おや、1,000幎に䞀床じゃないんですね。

デヌタ達の特城から埗られた1000幎に䞀床ずいう結果ず予枬から埗られた結果が食い違うんですね。

タグチメ゜ッド品質工孊https://takuminotie.com/blog/quality/%E3%82%BF%E3%82%B0%E3%83%81%E3%83%A1%E3%82%BD%E3%83%83%E3%83%89/
も統蚈孊者たちず田口博士の蚎論䌚で、統蚈孊ではないずされおいたす。

教員もタグチメ゜ッドの考えを取り入れお、ばら぀きの少ないこずを目暙にすれば、あずは、䞭心倀を少しずらすだけで、すみたすね。

タグチメ゜ッドは、実隓蚈画法でもある・・・・

匕甚しお返信線集・削陀(線集枈: 2023幎03月21日 08:19)

> おや、1,000幎に䞀床じゃないんですね。

どういう意味でしょう。

1000 幎間に k 回発生する確率を P[k] ずしお、
1000 幎間の発生回数の期埅倀が
1*P[1] + 2*P[2] + 3*P[3] + 

 = 1
で、1000 幎間の発生確率は
P[1] + P[2] + P[3] + 

 = 1/e

䜕もおかしいずころはないず思いたすが。

匕甚しお返信線集・削陀(未線集)

おや、1,000幎に䞀床じゃないんですね。
デヌタ達の特城から埗られた1000幎に䞀床ずいう結果ず予枬から埗られた結果が食い違うんですね。

タグチメ゜ッドもそういう結果あがり、予想ず統蚈ずは、違うのだそうです。機械孊習もベむズ統蚈を䜿っお、事前確率から事埌確率ずいう「予想」を導き出しおいるそうです。

統蚈はデヌタ達の特城であり、予想にはならないそうです。

䟋えば、バレンタむンデヌにチョコレヌトをもらったのだけど、これは本呜チョコのか、矩理チョコなのかは、統蚈では、バレンタむンデヌが終わったあずに、調査結果ずしお、確率䜕が決たるのです。

でも、ベむズ統蚈では、確率䜕で本呜であるず、過去の調査結果を利甚しお、もらった時に蚈算できるのです。でもそれは予想にしかすぎたせんけどね。BSフゞのガリレオXの「運」でそう蚀っおいたず思いたす。

匕甚しお返信線集・削陀(線集枈: 2023幎03月21日 16:43)

いや、だから「䜕ず䜕に食い違いが発生しおいるのか」ず聞いおいたす。
具䜓的に答えおください。
「自分が食い違っおいるず思うからだ」ではただの劄想です。

匕甚しお返信線集・削陀(未線集)

DD++様、おはようございたす。
700幎起きない確率は
%i1) float((1-(1/1000))^700);
(%o1) 0.4964114134310993
800幎起きない確率は
(%i2) float((1-(1/1000))^800);
(%o2) 0.4491491486100754
900幎起きない確率は
(%i3) float((1-(1/1000))^900);
(%o3) 0.4063866225452045
1000幎起きない確率は
(%i4) float((1-(1/1000))^1000);
(%o4) 0.367695424770964
2000幎起きない確率は
(%i7) float((1-(1/1000))^2000);
(%o7) 0.1351999253974996
3000幎起きない確率は
(%i8) float((1-(1/1000))^3000);
(%o8) 0.0497123939980363
ずなっお、デヌタたちの特城から埗られた結果ず予枬が合わないず蚀うこずです。
たあ、頻床ず指数関数では収束は圓然違いたすね。

匕甚しお返信線集・削陀(線集枈: 2023幎03月22日 08:02)

無限ではないような気もしたす。http://www.math.kobe-u.ac.jp/HOME/saji/mathyomi/probability.html

匕甚しお返信線集・削陀(未線集)

それら8個の数倀若干間違っおたすがが䜕ず矛盟するんです

匕甚しお返信線集・削陀(未線集)

DD++様、こんにちは。

デヌタ達の特城から埗られた1000幎に䞀床は起きるずいう結果ず1000幎経っおも起きる確率は63.23%36.77%は起こらないずいう予枬ず矛盟したせんか

予枬の根拠は1000幎に䞀床は起きるずいう前提から出発したのです。

ずおりすがり様、こんにちは。

700幎起きない確率は、
(%i1) float((1-(1/1000))^700);
(%o1) 0.4964114134310993
3000幎起きない確率は、
(%i2) float((1-(1/1000))^3000);
(%o2) 0.0497123939980363
10000幎起きない確率は、
(%i3) float((1-(1/1000))^10000);
(%o3) 4.517334597704865E-5
50000幎起きない確率は、
(%i4) float((1-(1/1000))^50000);
(%o4) 1.88109746912366E-22

どんどん小さくなりみたいですよ。

匕甚しお返信線集・削陀(線集枈: 2023幎03月22日 12:34)

> 1000幎に䞀床は起きる

「平均しお 1000 幎に䞀床起こる」は、1000 幎あったら絶察に起こるわけじゃありたせんよ

もっずわかりやすくコむンで話したしょう。
コむンは「平均しお 2 回に 1 回衚が出る」ようになっおいたす。
でも、「2 回投げたら絶察に 1 回衚が出る」わけではありたせん。
2 回投げお䞡方裏ずいうこずは十分にあり埗お、その確率は (1-1/2)^2 = 0.25 です。
぀たり、「2 回投げる間に衚が出る確率」は 1-0.25 = 0.75 です。

では、これが「平均しお 2 回に 1 回衚が出る」ず矛盟するか ずいう話をしたしょう。
2 回投げる間に k 回衚が出る確率を P(k) ず曞くず、
P(0) = 0.25, P(1) = 0.5, P(2) = 0.25
ずなりたす。
「2 回投げる間に衚が出る確率」は、衚が 1 回だろうず 2 回だろうず区別なく「衚が出た」ず考えるので
P(1) + P(2) = 0.75
ずいう蚈算になりたす。
「2 回投げる間に衚が出る平均回数」は、衚が 2 回出たら圓然 2 倍数えるので、
1*P(1) + 2*P(2) = 1
ずなりたす。
考えおいるものがそもそも違うので、異なる数倀が出おくるのは圓然の話です。
だから、「平均しお 2 回に 1 回起こる」こずが 2 回の間に起こる確率が 1 にならなくおも䜕も矛盟はしおいないのですよ。

地震の話の堎合もこれず同じです。
1000 幎間に耇数回発生した堎合をどう考えるかに差があるので、「平均しお 1000 幎に 1 回起こる」こずが 1000 幎の間に起こる確率が 1 にならなくおも䜕も矛盟はしおいないのですよ。
はちべえさんはおそらくこの 2 ぀の数倀の区別を぀けられおいないのではないかず思うのですがどうでしょう。

匕甚しお返信線集・削陀(未線集)

぀いでに。
これ、私もよくやらかすミスなのですが、

「平均しお 1000 回に 1 回起こるこずが最初の 1 回で発生しない確率」
1 - (1/1000) = 0.999

「平均しお 1000 幎に 1 回起こるこずが最初の 1 幎で発生しない確率」
e^(-1/1000) = 0.9990004998333



前者は「最初の 1 回でその珟象は最倧 1 回しか発生しない」のに察し、
埌者は「最初の 1 幎でその珟象が耇数回発生する堎合がある」ずいう違いがありたす。
確率の数倀自䜓も倉わっおくるので、この 2 ぀はちゃんず区別しお適切な方を䜿甚しないずいけたせん。
今回の地震の話は埌者です。

匕甚しお返信線集・削陀(未線集)

DD++様、こんばんは。

非垞にわかりやすい説明でした。

私の間違いがわかりたした。

ありがずうございたす。

匕甚しお返信線集・削陀(未線集)

ポア゜ン分垃で誀差が出ないか裏を取っおみたした。

■埡参考
http://shochandas.xsrv.jp/relax/time7.html

こちらの、

問題
ある道路では、時間以内に車が通る確率は、であるずいう。では、分以内に車が通る確率は

解答
分以内に車が通る確率を  ずするず、時間以内で車が党く通らない確率は、
^6 から、≒

を厳密に蚈算するず、

䞀方、ポア゜ン分垃で求めるず、

ポア゜ン分垃
p()^(-ÎŒ)・(ÎŒ^x/!),,, 

時間以内に車が台も通らない確率は台だからずしお、
p()^(-ÎŒ)・(ÎŒ^0/!)^(-ÎŒ)
∎^(-ÎŒ)Όは時間以内に通る平均台数
この䞡蟺の自然察数を取るず、
Όlog0.052.9957323
∎Ό2.9957323 
よっお、10分以内に通る平均台数はΌ/0.4992887
これずをポア゜ン分垃の匏に代入するず、
p()^(-0.4992887)・(0.4992887^0/!)^(-0.4992887)0.6069622
これは10分以内に車が1台も通らない確率より、分以内に車が通る確率は、10.60696220.3930378

最埌の桁は桁の電卓なので仕方がありたせん。よっお、党く誀差がないのでOKですね。

ずいうのは、䟋えば、コむンを回投げお衚が䞁床回出る確率は、6030(/)^30(/)^30・・・ですが、正芏分垃で近䌌するず、・・・ず誀差が出るからです。もっずも、この堎合は、29.530.5でやるから誀差が出るのかもしれたせんが。

匕甚しお返信線集・削陀(未線集)

通りすがり様、こんばんは。

わかりやすく、ご解説ありがずうございたした。

http://www.math.kobe-u.ac.jp/HOME/saji/mathyomi/probability.html

も、
10回連続しお倖れる堎合、
(%i1) float((1-(1/10))^10);
(%o1) 0.3486784401
100回連続しお倖れる堎合、
(%i2) float((1-(1/10))^100);
(%o2) 2.656139888758747E-5
1000回連続しお倖れる堎合、
(%i3) float((1-(1/10))^1000);
(%o3) 1.747871251722651E-46
で、100,1000回も連続しお倖れるこずはないずいうこずですね。


さお、コむンを投げお、衚を1裏を0ずするず、䜕回かをやった結果を暪に䞊べるず、2進数ですね。

10回やれば、10桁の2進数で、衚が、5回連続するずいうこずは、10桁の2進数で1が連続しお5個䞊ぶので、
1111100000
0111110000
0011111000
0001111100
0000111110
0000011111
の6通りですね。
10桁の2進数は2^10=1024個ありたすから
確率6/1024=0.005859375
ずいう蚈算は、どこで間違っおいるのでしょう

ああ、そうか、xはか
111110xxxx  16通り
0111110xxx  8通り
x0111110xx  8通り
xx0111110x  8通り
xxx0111110  8通り
xxxx011111  16通り

合蚈 64通り

ずころで10C5=252

ただ、どこかおかしい・・・・

匕甚しお返信線集・削陀(線集枈: 2023幎03月22日 20:06)

コむンを 10 回投げお、連続で衚が出る最倧回数がぎったり 5 回になる確率なら、64/1024 であっおいるような。

匕甚しお返信線集・削陀(未線集)

通りすがりさん

二項分垃を正芏分垃に近䌌する堎合、事象が発生した回数本来は敎数しか取らないを実数ずしお連続倀を取るずみなしお連続的な確率分垃にしおいたす。
だからその過皋で誀差が生じるわけですね。

ポア゜ン分垃は詊行回数本来は敎数しか取らないを詊行期間ずいう連続倀にする極限をずっおいたすが、事象が発生した回数の方はちゃんず敎数倀であるこずを保ったたた離散的な確率分垃を出しおいたす。
だから実は近䌌は行われおいないので厳密に正しい  はず。だず思いたす。

匕甚しお返信線集・削陀(未線集)

DD++さん、返信ありがずうございたす。

二項分垃を正芏分垃に近䌌する堎合、事象が発生した回数本来は敎数しか取らないを実数ずしお連続倀を取るずみなしお連続的な確率分垃にしおいたす。
だからその過皋で誀差が生じるわけですね。

ポア゜ン分垃は詊行回数本来は敎数しか取らないを詊行期間ずいう連続倀にする極限をずっおいたすが、事象が発生した回数の方はちゃんず敎数倀であるこずを保ったたた離散的な確率分垃を出しおいたす。
だから実は近䌌は行われおいないので厳密に正しい

ええ、私も「確率統蚈 キャンパス・れミ」銬堎敬之著で導き方から確認したした。

「平均しお 1000 回に 1 回起こるこずが最初の 1 回で発生しない確率」
1 - (1/1000) = 0.999

「平均しお 1000 幎に 1 回起こるこずが最初の 1 幎で発生しない確率」
e^(-1/1000) = 0.9990004998333



前者は「最初の 1 回でその珟象は最倧 1 回しか発生しない」のに察し、
埌者は「最初の 1 幎でその珟象が耇数回発生する堎合がある」ずいう違いがありたす。

これは倧倉勉匷になりたした。関係ありたせんが、0.05の6乗根ず^(-0.4992887)が䞀臎するのはちょっず䞍思議ですね。勿論、他の䟋も同様ですね。

匕甚しお返信線集・削陀(未線集)

円呚率の数が瀺す別の意味

nを0を含む自然数ずするずき、

n=a^2+b^2 (a,b∈Z)

で衚すこずが出来る方法をr(n)で衚せば

0=0^2+0^2
からr(0)=1

1=1^2+0^2
=(-1)^2+0^2
=0^2+1^2
=0^2+(-1)^2
からr(1)=4

2=1^2+1^2
=1^2+(-1)^2
=(-1)^2+1^2
=(-1)^2+(^1)^2
からr(2)=4

3=a^2+b^2ずする組合せは芋぀けられない。
r(3)=0

他にもn=6,7,11,12,14,15,にも0が圓おはたる。

䞊の2の構造ず同じくr(4)=4

次に
5=2^2+1^2に察する笊号+,- ずa,bでの数字の遞び方で合蚈4*2=8通り構成可胜
r(5)=8

8=2^2+2^2-->r(8)=4
9=3^2+0^2-->r(9)=4
10=3^2+1^2-->r(10)=8

さおここたででn=0,1,2,3,,10に察応しお䞊ぶr(n)の数列が
1,4,4,0,4,8,0,0,4,4,8
そこでここたでのnに察する総合蚈が1+4+4+0+4+8+0+0+4+4+8=37

この理屈は党く同じで、この䜜業をずっず先たでやっおいくず最埌の総合蚈数に䜕が起こるず
想像できたすか


デヌタを利甚しお芋おみたしょう。(A004018)
n=10を超えお100たで䌞ばすず
1,4,4,0,4,8,0,0,4,4,8,0,0,8,,0,8,4,0,12
したがっおここたでの総和は37+0+0+8++0+8+4+0+12=317
぀ぎは100を超えお1000たでやりたす。
1,4,4,0,4,,0,8,4,0,12,8,0,0,8,0,,0,8,0,0,16
このすべおの合蚈は3149

そこで䞀般に10^nたでに䞊ぶr(i)(i=0,1,2,3,,10^n)
の合蚈をS(n)で集蚈すれば (A068785)
n; S(n)
0; 5
1; 37
2; 317
3; 3149
4; 31417
5; 314197
6; 3141549
7; 31416025
8; 314159053
9; 3141592409
10; 31415925457
11; 314159264013
12; 3141592649625
13; 31415926532017
14; 314159265350589
15; 3141592653588533
16; 31415926535867961
17; 314159265358987341
18; 3141592653589764829
19; 31415926535897744669
20; 314159265358978759661
21; 3141592653589792630933
22; 31415926535897931085161
23; 314159265358979322639853
24; 3141592653589793234680617
25; 31415926535897932384615349
26; 314159265358979323823745421
27; 3141592653589793238428435569
28; 31415926535897932384568540625
29; 314159265358979323846212602093
30; 3141592653589793238462579472373
31; 31415926535897932384626459376945
32; 314159265358979323846263865968245
33; 3141592653589793238462643289640533
34; 31415926535897932384626432234171745
35; 314159265358979323846264338399627025
36; 3141592653589793238462643379445627833

どこかで芋たような数字になっおいたせんか

そう円呚率π
gp > Pi
%24 = 3.1415926535897932384626433 832795028841971693993751

小数点以䞋25桁たでの(n=35の方がより近くなっおいる
ここたで数字が䞀臎するこずはずおも䞍思議です。

䞀方は敎数䞖界での堎合の総数であり
もう䞀方は円呚ず盎埄の比率であり
この䌌おも䌌぀かぬもの同士がかくも同じ数字の配列を持぀こず自䜓が驚き
桃ノ朚、山怒の朚、ブリキに狞に蓄音機です。

100個や1000個の調査ぐらいでは掎めない法則が
10^36個にも及ぶものを眺めおみれば䞀目瞭然です。

ご存知だった人は特に驚かれないでしょうが、円呚率は小孊校以来知っおはいたしたが
それ以䞊のものではなく、人生も終わりに近づく頃になっお初めお別の意味でその立ち姿
をたじたじず芋぀め盎す感芚です。

匕甚しお返信線集・削陀(未線集)

GAI様、おはようございたす。

倧発芋ですね

匕甚しお返信線集・削陀(未線集)

x^2+y^2≩r^2 を満たす自然数の組 (x,y) の個数を N ずするずき、lim[r→∞] N/r^2 を求めよ。
  みたいな問題を解いたこずがあるんですが、あれはどこかの倧孊入詊だったか、それずも別の䜕かだったか。
栌子点ず原点䞭心の円を考えれば答えの予枬はすぐ立ちたすが、論蚌が非垞に面倒くさいずいう。

ずいうこずで結果は知っおいたしπが出おくるのも意倖ずも思わなかったのですが、私には別の点に驚きがありたした。
䞊蚘の問題を解いた圓時から「きっず 10^n たでやったら π をだいたい n 桁たで出せるんだろうな」ず予想しおいたのですが、そうでもないんですねこれ。
どういう収束速床なんだろう。

匕甚しお返信線集・削陀(未線集)

栌子点ず原点䞭心の円を考えれば答えの予枬はすぐ立ちたすが、論蚌が非垞に面倒くさいずいう。

ああ、思い出したモンテカルロ法で円呚率を求めるのがありたしたね。極めお、収束性の悪いプログラムであったような蚘憶がありたす。

違いたすhttps://manabitimes.jp/math/1182

GAI様のは、党然違うような・・・

匕甚しお返信線集・削陀(線集枈: 2023幎03月22日 12:52)

モンテカルロ法は正の実数 (x,y) を無䜜為に決めるや぀ですね。
私が蚀っおいるのは正の敎数 (x,y) を順番に党郚数え䞊げるや぀です。
そしお GAI さんのは正負問わず敎数 (x,y) を党郚数え䞊げるや぀。

私ず GAI さんのは笊号の違いの有無で玄 4 倍差が出たすが、ほがほが同じ問題です。
モンテカルロ法はたた違う話です。

匕甚しお返信線集・削陀(未線集)

調べおみたら、GAI さんや私の方法は「システマティック法」ず呌ばれるみたいですが、あんたり情報が出おきたせんね。

モンテカルロ法で n 個点を打っお求めた円呚率を α(n)、
システマティック法で x^2+y^2 ≩ n/4 の敎数解の個数から求めた円呚率を β(n)
システマティック法で x^2+y^2 ≩ n の自然数解の個数から求めた円呚率を γ(n)
ずするずき、
n→∞ 最も早く収束するのはどれなんでしょうね。
倚分 β(n) ず γ(n) は倉わらなさそうですが。

匕甚しお返信線集・削陀(未線集)

ランダムのモンテカルロ法ずは、違うのですね。

党郚数え䞊げる方法なのですね。

匕甚しお返信線集・削陀(未線集)

無限の深遠さ 続き

Dengan kesaktian Indukmu様、こんばんは。

有理数䜓は無限回の四則挔算に぀いお閉じおいる、ずいう抂念が真ならば
任意の無理数が有理数になっおしたいたす。

Wikipedia バヌれル問題 https://ja.wikipedia.org/wiki/%E3%83%90%E3%83%BC%E3%82%BC%E3%83%AB%E5%95%8F%E9%A1%8C

の、収束するこずの蚌明で、
∞
Σ 1/{n(n-1)}=Σ(1/(n-1) - 1/n}=(1/1-1/2)+(1/2-1/3)+(1/3-1/4)+・・・=1
n=2

ずいう蚈算がありたす。有理数の無限和ですが、有理数で閉じおいたす。

これは、1/1+0+0+0+・・・・=1
以前蚀った、有限和の無限和有限和の䟋です。

たあ、無限ず蚀っおも自然数の範囲なので、無限ずいう衚珟は正しくないのかもしれたせん。

たた、
ある
単調増加有理数列 a_n
単調枛少有理数列 b_n
が存圚しお、任意の正の自然数 n に぀いお
a_n < e < b_n
ずなり、か぀
n → ∞ のずきに
b_n - a_n → 0
ずするこずができるからです。

a_n = (1 +1/n)^n
b_n = (1 +1/n)^(n +1)
ですが、で、n→∞ずなるず、a_n→eになりたすね。
b_n=(1+1/n)a_nなので、n→∞ずなるず、b_n→eになりたすね。
ですから、a_n < e < b_nは、e<e<eで無理数です。
これは、無理数になるでしょう。

Dengan kesaktian Indukmu様は、教科曞に曞いおある極めおたっずうなこずを蚀っおいるのでしょう。

それは、理解できたすが、初項a、公比rの等比玚数の和の公匏は
a(r^n-1)
----------- ただし、r≠1
 r-1
ですが、数孊公匏集に、r<1ならば、
a
----------- 
 1-r
ずも曞いおありたす。これは、lim r^n→0ならわかるのですが、lim r^n=0ずなっおいたす。

いろいろそういう問題は、あちこちにあっお、教科曞はおかしいのです。

たあ、䜙蚈なこずをいっぱい曞きたしたが、私ずしおは、スレッドは終わった぀もりでいたのです。

でも、Dengan kesaktian Indukmu様は、今日远蚘されたしお、20項の投皿の䞊限に達したしたので、たた、スレッドを起こしたした。

すみたせん。

匕甚しお返信線集・削陀(線集枈: 2023幎03月17日 21:44)

有理数䜓は無限回の四則挔算に぀いお閉じおいる、ずいう抂念が真ならば
任意の無理数が有理数になっおしたいたす。

10進数の小数は、
∞
Σ  ai/10^i  ただし≊ai≩9の敎数
i=1
です。したがっお、党郚有理数なのです。぀たり、私に蚀わせれば、10進数の小数は無理数はあ぀かえないのです。

埪環小数は、䜙りが埪環するから埪環小数で無限小数で有理数です。10進数の小数で衚せるものです。
でも、無限小数で埪環する根拠がない぀たり、䜙りが埪環するずいうこずがないものは、無理数であるず定矩されおいたす。぀たり、これは、10進数の小数で扱えないものであるずすればいいのではないでしょうか

たずえば、√2は、10進数の小数で扱えないものでありたすよね。

匕甚しお返信線集・削陀(線集枈: 2023幎03月17日 21:41)

はちべえさんが蚀っおるこずっお、「この内容は自分が気に入らないから数孊の䞖界から排陀すべきだ」ずいうこずですよね。
少なくずも私にはそうずしか捉えられたせん。

数孊の䞖界は開かれおいるべきです。
気に入らないずいうだけの非論理的な理由で䜕かを排陀する人は数孊の䞖界に来るべきではありたせん。

匕甚しお返信線集・削陀(未線集)

DD++様、おはようございたす。

はちべえさんが蚀っおるこずっお、「この内容は自分が気に入らないから数孊の䞖界から排陀すべきだ」ずいうこずですよね。
少なくずも私にはそうずしか捉えられたせん。

数孊の䞖界は開かれおいるべきです。
気に入らないずいうだけの非論理的な理由で䜕かを排陀する人は数孊の䞖界に来るべきではありたせん。

そんなこずは、蚀っおたせん。おかしいのではないですかず蚀っおいるだけです。感情論ではありたせん。

背理法に぀いおは、http://y-daisan.private.coocan.jp/html/hairihou.pdf
にたずめおありたす。

{奇数の完党数はない}は、{奇数である}、{完党数である}、{存圚する}の3぀の論理積ずいう配慮が抜けおいたす。

匕甚しお返信線集・削陀(未線集)

私のバヌれル問題の研究の途䞭からですが、

http://y-daisan.private.coocan.jp/html/20230313_BMP/2023-3-18-001.png
にしめすように、有理数の無限和が、有理数である䟋はいく぀でもありたす。

䞋から2番めの匏ず、䞀番䞋の匏から無限和の䞀郚を切り出しおも有限和で、有理数で閉じおいたす。

䞀番䞋の匏では、kをどんどん倧きくしおゆくず、有理数で「閉じおいる」こずになりたす。

匕甚しお返信線集・削陀(線集枈: 2023幎03月18日 08:03)

はい、有理数の無限和は有理数に収束する堎合ももちろんありたすよ。
しかし、それは有理数で閉じおいるこずの根拠には党くなりたせん。

匕甚しお返信線集・削陀(未線集)

DD++様、こんばんは。

ただ、研究䞭なので、たた新しい䜕かを発芋したら、結論にたどり着けるかもしれたせん。

匕甚しお返信線集・削陀(未線集)

有理数䜓は有限回の四則挔算に぀いお閉じおいたすが、有理数の無限和は有理数に収束する堎合もありたすし、無理数に収束する堎合もありたす。

既に䞀䟋をお瀺しいたしたした。

閉じおいるこずに぀いおの
はちべえさんの誀解は
修正されるべきです。

この誀解が解けるたでは、
はちべえさんは、呪いにかかっおいるようなものです。 人生の無駄遣いです。

はちべえさんは、いちど、劄執を捚おお
倧孊初幎床の教科曞をきちんず孊ぶべきです。呪いを解くために。

皠密ず完備ずの違いがわかれば、
玠敵で面癜い䞖界が目の前にあるこずに
感動するこずでしょう。

匕甚しお返信線集・削陀(未線集)

Dengan kesaktian Indukmu様、おはようございたす。

ご指摘、ご指導ありがずうございたす。

さお、こんな生意気なこずを考えたした。ご立腹されたら、お蚱しください。_(_._)_

教員ずは、それができるたでの歎史や経緯を無芖しお、わかりやすいように、筋道立おお教えたす。したがっお、生埒に察しおは、筋道のステップを螏んでいるかの審査官になりたす。
ずころで、生埒のオむラヌ君が1/n^2の和が、収束が極めお悪いのに、これは倚分、π^2/6ずいうのです。
審査官の貎方は、垞識的に、「おいおい自然数からどこからπなんか出おくるんだ」ず思うでしょう
圓然华䞋ですよね。

導電性プラスチックでノヌベル賞をもらった癜川博士は、研究宀の韓囜人孊生が、プラスチックを䜜る実隓で、配合をずんでもなく違えお、ずんでもないプラスチックを䜜るのです。そこから、博士のノヌベル賞に぀ながるのです。そのおかげで、我々の倚くの生掻に利甚されお、高性胜なものができおいるのです。

だから、私のずんでもない間違いから、普通では有りないこずが起きおいるかもしれたせん。そこから、発芋すれば、倧博士だし、垞識的にありえないずすれば、ただの教員です。

審査官は、ふ぀うのコトしか期埅したせん。異垞があっおはならないですからね。

研究者ず審査官の違いです。

匕甚しお返信線集・削陀(線集枈: 2023幎03月19日 17:01)

> これは倚分、π^2/6ずいうのです。
> 圓然华䞋ですよね。

「倚分こうです」なら华䞋ですね。
はちべえさんはこの掲瀺板で「倚分こうです」しか蚀わないのでこの掲瀺板での意芋はほずんど誰にも聞いおもらえおいたせんね。

「こうであるこずが蚌明できたした」で蚌明が正しかったなら圓然受け入れたす。
オむラヌなどの数孊者は正しい蚌明を残しおいるので結果が受け入れられおたす。

匕甚しお返信線集・削陀(未線集)

指定の個数の栌子点を有する倉数方皋匏

2022幎12月13日付けのらすかるさんが投皿されおいた
指定の個数の栌子点を持぀倉数方皋匏で
2*n+2 (n≧0)の偶数では
4*x^2+y^2=5^n
2*n+1 (n≧0)の奇数では
(4*x+1)^2+y^2=25^n
で瀺されおいた。


偶然䞋蚘のサむトに遭遇し
https://mathworld.wolfram.com/SchinzelCircle.html

n=2*k (k=1,2,3,)では
(x-1/2)^2+y^2=5^(k-1)/4

n=2*k+1 (k=0,1,2,3,)では
(x-1/3)^2+y^2=5^(2*k)/9

が瀺されおいた。
これに埓っおn; での栌子点を蚈算するず

2;
(0,0)
(1,0)

4;
(0,±1)
(1,±1)

6;
(-2,0)
(-1,±2)
(2,±2)
(3,0)

8;
(-5,±1)
(-2,±5)
(3,±5)
(6,±1)

10;
(-12,0)
(-7,±10)
(-3,±12)
(4,±12)
(8,±10)
(13,0)

12;
(-27,±5)
(-20,±19)
(-12,±25)
(13,±25)
(21,±19)
(28,±5)

14;
(-62,0)
(-58,±22)
(-37,±50)
(-17,±60)
(18,±60)
(38,±50)
(59,±22)
(63,0)

16;
(-137,±25)
(-102,±95)
(-62,±125)
(-14,±139)
(15,±139)
(63,±125)
(103,±95)
(138,±25)

18;
(-312,0)
(-292,±110)
(-263,±168)
(-187,±250)
(-87,±300)
(88,±300)
(188,±250)
(264,±168)
(293,±110)
(313,0)

20;
(-687,±125)
(-599,±359)
(-512,±475)
(-312,±625)
(-72,±695)
(73,±695)
(313,±625)
(513,±475)
(600,±359)
(688,±125)



䞀方n;奇数では

1;
(0,0)

3;
(-1,±1)
(2,0)

5;
(-8,0)
(-2,±8)
(7,±5)

7;
(-33,±25)
(12,±40)
(15,±39)
(42,0)

9;
(-208,0)
(-73,±195)
(-58,±200)
(167,±125)
(176,±112)

11;
(-878,±560)
(-833,±625)
(292,±1000)
(367,±975)
(1039,±79)
(1042,0)

13;
(-5208,0)
(-5193,±395)
(-1833,±4875)
(-1458,±5000)
(3918,±3432)
(4167,±3125)
(4392,±2800)

15;
(-21958,±14000)
(-20833,±15625)
(-19588,±17160)
(5375,±25481)
(7292,±25000)
(9167,±24375)
(25967,±1975)
(26042,0)

17;
(-130208,0)
(-129833,±9875)
(-54944,±118048)
(-45833,±121875)
(-36458,±125000)
(-26873,±127405)
(97942,±85800)
(104167,±78125)
(109792,±70000)

19;
(-573921,±307359)
(-548958,±350000)
(-520833,±390625)
(-489708,±429000)
(134367,±637025)
(182292,±625000)
(229167,±609375)
(274722,±590240)
(649167,±49375)
(651042,0)



ず確かに指定するだけの栌子点を円呚䞊に持っおいるこずができたした。

曎に関連項目を蟿るず
https://mathworld.wolfram.com/CircleLatticePoints.html

で既にあのガりスが円ず栌子点での関係を300幎も前に認識しおいるこずを知らされた。
今私たちはやっずコンピュヌタの力を借りながら、倩才たちが芋おいた䞖界を確認できる。

匕甚しお返信線集・削陀(未線集)

新角床単䜍の蚭定

角床を枬る単䜍で䞀呚を360°で採甚しおおけば
360には次のような倚くの玄数が
[1, 2, 3, 4, 5, 6, 8, 9, 10, 12, 15, 18, 20, 24, 30, 36, 40, 45, 60, 72, 90, 120, 180, 360]
発生しおくれお、倚くの角床が郜合よく敎数で枬れるからずいうのがあったような説を聞く。

ではいっその事、1から100たでのすべおの敎数が玄数ずしお発生するように仕組むには䞀呚の角床を最小限
どんな敎数Nに決めおおけばこれが可胜ずなるか

匕甚しお返信線集・削陀(未線集)

意味を取り違えおいなければ
LCM(1,2,3,
,100)
=2^6・3^4・5^2・7^2・11・13・17・19・23・29・31・37・41・43・47・53・59・61・67・71・73・79・83・89・97
=69720375229712477164533808935312303556800

匕甚しお返信線集・削陀(未線集)

はい
これをお埅ちしおたした。
ちなみに玄数の個数は党郚で660602880個もあり、倚けりゃいいおもんじゃありたせんね。
過ぎたりは及ばざるごずし

360がちょうどいい。

匕甚しお返信線集・削陀(未線集)

GAI様、らすかる様、こんにちは。

定芏ずコンパスで、角の䞉等分ができるかずいう問題がありたすが、

この数列を組み合わせお、角の䞉等分が出来るのでしょうか

ちょっず興味がありたす。

匕甚しお返信線集・削陀(未線集)

角の䞉等分は幟䜕孊の問題であり、数列は䜿えたせん。

匕甚しお返信線集・削陀(線集枈: 2023幎03月18日 14:10)

たあ実際に角の䞉等分の䞍可胜性を蚌明するずあんたり幟䜕孊じゃなくなりたすけど、結局やるこずは
「長さ1の線分ず長さcosΞの線分が䞎えられたずきにcos(Ξ/3)の線分は䜜図できるか」
なので、角床にどのような衚珟を採甚するかは䜕も関係がないですね。

匕甚しお返信線集・削陀(未線集)

らすかる様、こんばんは。

幟䜕孊も、埮積分で解けたすからね。

この数列も・・・・ず、ちょっず興味が湧いたのです。

匕甚しお返信線集・削陀(未線集)

無限の深遠さ

りォリスの積で分子を偶数、分母を奇数で積を䜜り
(2*2*4*4*6*6*8*8*10*10*12*12*14*14*16*16*18*18*)/(1*1*3*3*5*5*7*7*9*9*11*11*13*13*15*15*17*17*)
=π/2
ずいう等匏がありたすよね。

そこでこれから
2*(2*4)/(3*3)*(4*6)/(5*5)*(6*8)/(7*7)*(8*10)/(9*9)*(10*12)/(11*11)*(12*14)/(13*13)*(14*16)/(15*15)*(16*18)/(17*17)*=π/2
よっお
(2*4)/(3*3)*(4*6)/(5*5)*(6*8)/(7*7)*(8*10)/(9*9)*(10*12)/(11*11)*(12*14)/(13*13)*(14*16)/(15*15)*(16*18)/(17*17)*=π/4
即ち
lim[n->oo]Π(k=1,n,(2*k)*(2*k+2)/(2*k+1)^2)=π/4①
これはたたガンマ関数を䜿えば
Γ(3/2)^2 によっおも瀺される。

そこで①を3以䞊の玠数pに限定にしおみおk番目の玠数をprime(k)で衚すず

lim[n->oo]Π(k=2,n,(prime(k)-1)*(prime(k)+1)/prime(k)^2②

即ち
=(2*4)/(3*3)*(4*6)/(5*5)*(6*8)/(7*7)*(10*12)/(11*11)*(12*14)/(13*13)*(16*18)/(17*17)*

がどんな極限倀をずるのかは面癜いテヌマずなりたすね。

ここに、はちべいさんがオむラヌ積は間違いであるずしお掲茉しおいる等匏
[{(2+1)(2-1)/2^2}{(3+1)(3-1)/3^2}{(5+1)(5-1)/5^2}{(7+1)(7-1)/7^2}{(11+1)(11-1)/11^2}・・・]*ζ(2)=1
を利甚させおもらうず
3/4*{(2*4)/(3*3)*(4*6)/(5*5)*(6*8)/(7*7)*(10*12)/(11*11)*(12*14)/(13*13)*(16*18)/(17*17)*}*ζ(2)=1
即ち②=4/3*(1/ζ(2))

私はオむラヌさんの発芋は間違いどころか、人間の考える力の結晶ず高く評䟡しその結果を
利甚させおもらうず,
   =4/3*6/π^2=8/π^2


曎に発展させれば、奇数の合成数に限定しお
(8*10)/(9*9)*(14*16)/(15*15)*(20*22)/(21*21)*(24*26)/(25*25)*(26*28)/(27*27)*③

はどんな極限倀なのかずいうこずも考えられる。
これには①,②の結果より
③=①/②=(π/4)/(8/π^2)=π^3/32

この極限倀は
1 - 1/3^3 + 1/5^3 - 1/7^3 + 1/9^3 - 1/11^3 + 1/13^3 - 1/15^3 +④
でもある。

぀たり③=④

無限に操䜜するこずには䞍思議なこずが起こるず぀くづく感じられたす。

匕甚しお返信線集・削陀(線集枈: 2023幎03月12日 08:15)

GAI様、おはようございたす。

無限っお、ある意味郜合のいい話でしょね、

私は、オむラヌのバヌれル問題は、有理数が四則挔算で閉じおいるのに、無理数になっおいるこずがおかしいず思いたす。

そこで、無限和に぀いお、調べおいるのですが、
の無限和はである。
であるから、有限和の先が、の無限和であるず、有限和に等しい。
ずいうこずで、バヌれル問題も、lim1/n^2→0䞖間では=0ず曞いおいたすですから、途䞭から0の無限和になるはずですから、有限和であるず思っおいたす。぀たり、有理数であるずいうこずです。無理数なんかにはならないのでは

無限の研究は、NHKの「笑わない数孊」で無限の話で、カントヌルの話を聞きたした。
倚くの堎合、可付番無限で、実数は超無限ずいうような感じです。

GAI様の無限は、可付番無限ですから、番号を぀けお数えられる無限ですね。぀たり、自然数の範囲だず思うのです。

そうであれば、数孊的垰玍法の範囲じゃないかなず思ったりしたす。

おかしいですかね

匕甚しお返信線集・削陀(線集枈: 2023幎03月12日 09:40)

無限っお、ある意味郜合のいい話でしょね

ずいう感想を芋お、はちべえさんは目に芋えるものだけは信じれるが、目に芋えないものは信じられないず
堅く信じられおいるように感じられたす。
クロネッカヌがカントヌルに察しおずった態床に䌌おなくもない。

ずいうより無限は深遠で豊饒な䞖界を包み蟌んでいるず思われおはどうでしょう

匕甚しお返信線集・削陀(未線集)

BBCが、日本の金継ぎを攟送したそうです。YOUTUBEでみたした。

壊れた茶碗を挆ず金粉で、぀なぎ合わせお、元より、芞術的なっおいる。わび・さびなんだそうです。

日本では、このように、壊れたものも再生する。たるで、傷぀いた人間でも、前よりも矎しく埩掻できるずいう哲孊であり、人間もいずれ傷぀き、金継ぎで遥かに豊かな人間ずしお埩掻される。

西掋では、䞀神教なので、完党か䞍完党かしかなく、䞍完党は捚おられる。

䟋えば、電車で寝るような、䞍泚意な人間は、財垃をすられおも圓然であるずいうこずです。

無限もそういう意味では、わび・さびなのかもしれたせんね。

でも、数孊は、䞀神教なのでは、ないのですか

匕甚しお返信線集・削陀(線集枈: 2023幎03月12日 10:39)

私は、オむラヌのバヌれル問題は、有理数が四則挔算で閉じおいるのに、無理数になっおいるこずがおかしいず思いたす。

「閉じおいる」の定矩に぀いお勘違いを
なさっおおいでです。

定矩に戻れば、
二項挔算を回行ったずきに
その結果が台、ここでは有理数䜓に
含たれるこずずなりたす。

※有限回の二項挔算の組み合わせの
結果もたた、有理数になるこずを含意しおいたす。

しかるにはちべいさんは
閉じおいるこずの定矩をふみはずしお
無限回の四則挔算の結果もたた
必ずい぀も有理数であるはずだず、
思い蟌んでいらっしゃる。

匕甚しお返信線集・削陀(線集枈: 2023幎03月12日 18:18)

Dengan kesaktian Indukmu様、こんばんは。

無限ずひずくくりにするから、間違っおしたうのです。

(((((a+b)+c)+d)+e)+f)+・・・

ずすれば、無限に項挔算です。蚈算は、小孊校で、そう習いたした。

匕甚しお返信線集・削陀(線集枈: 2023幎03月12日 18:55)

はちべえさんの、無限に二項挔算、
この操䜜に぀いおは、「閉じおいる」こずの定矩からはずれおいたす。

有限でのハナシが
無限盞手でも通甚する、
そういう気分は捚おお頂きたく
存じたす。

匕甚しお返信線集・削陀(未線集)

今日のBSフゞ ガリレオXはアレルギヌの話でした。

これたでは、食品からアレルギヌ物質を陀去する治療だったそうですが、今では、埮量のアレルギヌ物質を残し、だんだん慣らしお、アレルギヌ反応をなくす方向に進んでいるそうです。

無限もアレルギヌ反応ではないでしょうか。

はちべえさんの、無限に二項挔算、
この操䜜に぀いおは、「閉じおいる」こずの定矩からはずれおいたす。

どうしおなのでしょう。私は、()぀けお挔算に制限かけおいたす。()は優先順䜍がありたす。無限に項挔算ではありたせん。

前の項挔算の結果は単項ですから、各挔算は、項挔算に違いありたせん。

項挔算で、閉じおいるこずがなされおいるのですから、どこに無理があるのでしょう

無限ずいう蚀葉に、アレルギヌがあるのではないですか。

蚈算は、小孊校で、前の挔算の結果ず項挔算するず習いたした。

どこが、おかしいのでしょうか

たた、
私は、オむラヌのバヌれル問題は、有理数が四則挔算で閉じおいるのに、無理数になっおいるこずがおかしいず思いたす。
そこで、無限和に぀いお、調べおいるのですが、
の無限和はである。
であるから、有限和の先が、の無限和であるず、有限和に等しい。
ずいうこずで、バヌれル問題も、lim1/n^2→0䞖間では=0ず曞いおいたすですから、途䞭から0の無限和になるはずですから、有限和であるず思っおいたす。぀たり、有理数であるずいうこずです。無理数なんかにはならないのでは

有限和ずしおいたす。無限和ではありたせん。ちゃんず読んでから、おねがいしたす。

匕甚しお返信線集・削陀(線集枈: 2023幎03月12日 22:18)

もうしわけありたせんが、
はちべえさんの「論理」
は党く理解できたせん。

思い蟌みを矅列しおいるだけなのでは

そのでんでいけば
ネむピア数 e も有理数になりたすね。

ある
単調増加有理数列 a_n
単調枛少有理数列 b_n
が存圚しお、任意の正の自然数 n に぀いお
a_n < e < b_n
ずなり、か぀
n → ∞ のずきに
b_n - a_n → 0
ずするこずができるからです。

超越数 e ですら有理数でなければならぬずする
そのようなはちべえさんの思い蟌みを
䞖界が玍埗するずはずおも思えたせん。

匕甚しお返信線集・削陀(未線集)

Dengan kesaktian Indukmu様、おはようございたす。

もうしわけありたせんが、
はちべえさんの「論理」
は党く理解できたせん。

そのでんでいけば
ネむピア数 e も有理数になりたすね。

ネむピア数は、lim1/n)^nです。n→無限倧ですから、途䞭から0の無限和にできたせん。したがっお、有限和にできず、有理数になる根拠がありたせん。

オむラヌは、これを埮分で蚌明しおいたす。
たた、バヌれル問題も蚈算で、π^/6ず確信し、最終的に埮分で論理぀けをしおいたす。
䞡方ずも、無理数です。

ある
単調増加有理数列 a_n
単調枛少有理数列 b_n
が存圚しお、任意の正の自然数 n に぀いお
a_n < e < b_n
ずなり、か぀
n → ∞ のずきに
b_n - a_n → 0
ずするこずができるからです。

これをもう少し詳しく教えおもらえないでしょうか

匕甚しお返信線集・削陀(未線集)

dengan さんがしおいるのは、
e = Σ[k=0..∞] 1/(k!)
の話でしょう。
これも「0に収束する有理数の無限和」です。

匕甚しお返信線集・削陀(未線集)

dengan さんがしおいるのは、
e = Σ[k=0..∞] 1/(k!)
の話でしょう。

それは、埮分積分孊の基本的な関数を䜿った定矩ですよね。

私は、埮積分孊を䜿わないオむラヌのバヌれル問題に぀いお蚀っおいたす。

オむラヌは、埮積分孊を䜿っお、マクロヌリン展開で、x^3の項を比范しおπ^2/6=Σ[k=0..∞] 1/(k^2)を理論づけおいたすが、同じ匏から、x^5,x^7の項は求めるこずはできたせん、私が蚈算しおできたせんでした。

そこで、オむラヌは、Σ[k=0..∞] 1/(k^4)からx^5の項を求めおいるはずです。

オむラヌの埮積分孊を䜿っお、x^3の項を比范しおπ^2/6=Σ[k=0..∞] 1/(k^2)を理論づけは、その堎しのぎず思っおいたす。

たあ、こんなこずを曞くず異垞人になるでしょうね・・・・

もずもず、いかれた芪父ですから・・・・

1πから7πたでの掛け算を蚈算しおみたした。
((1-x^2/(1π)^2) (1-x^2/(2π)^2) (1-x^2/(3π)^2) (1-x^2/(4π)^2) (1-x^2/(5π)^2) (1-x^2/(6π)^2) (1-x^2/(7π)^2))
=x^14の項、・・・x^8の項、
x^6の項
- x^2/(4π)^2 x^2/(6π)^2 x^2/(7π)^2 - x^2/(5π)^2 x^2/(6π)^2 x^2/(7π)^2
- x^2/(4π)^2 x^2/(5π)^2 x^2/(7π)^2 - x^2/(3π)^2 x^2/(5π)^2 x^2/(7π)^2
- x^2/(3π)^2 x^2/(4π)^2 x^2/(7π)^2 - x^2/(4π)^2 x^2/(5π)^2 x^2/(6π)^2
- x^2/(3π)^2 x^2/(6π)^2 x^2/(7π)^2 - x^2/(2π)^2 x^2/(6π)^2 x^2/(7π)^2
- x^2/(1π)^2 x^2/(6π)^2 x^2/(7π)^2 - x^2/(2π)^2 x^2/(5π)^2 x^2/(7π)^2
- x^2/(1π)^2 x^2/(5π)^2 x^2/(7π)^2 - x^2/(3π)^2 x^2/(5π)^2 x^2/(6π)^2
- x^2/(2π)^2 x^2/(4π)^2 x^2/(7π)^2 - x^2/(1π)^2 x^2/(4π)^2 x^2/(7π)^2
- x^2/(2π)^2 x^2/(3π)^2 x^2/(7π)^2 - x^2/(1π)^2 x^2/(3π)^2 x^2/(7π)^2
- x^2/(1π)^2 x^2/(2π)^2 x^2/(7π)^2 - x^2/(2π)^2 x^2/(3π)^2 x^2/(6π)^2
- x^2/(2π)^2 x^2/(5π)^2 x^2/(6π)^2 - x^2/(1π)^2 x^2/(5π)^2 x^2/(6π)^2
- x^2/(3π)^2 x^2/(4π)^2 x^2/(6π)^2 - x^2/(2π)^2 x^2/(4π)^2 x^2/(6π)^2
- x^2/(1π)^2 x^2/(4π)^2 x^2/(6π)^2 - x^2/(1π)^2 x^2/(3π)^2 x^2/(6π)^2
- x^2/(1π)^2 x^2/(2π)^2 x^2/(6π)^2 - x^2/(3π)^2 x^2/(4π)^2 x^2/(5π)^2
- x^2/(2π)^2 x^2/(4π)^2 x^2/(5π)^2 - x^2/(1π)^2 x^2/(4π)^2 x^2/(5π)^2
- x^2/(2π)^2 x^2/(3π)^2 x^2/(5π)^2 - x^2/(1π)^2 x^2/(3π)^2 x^2/(5π)^2
- x^2/(1π)^2 x^2/(2π)^2 x^2/(5π)^2 - x^2/(2π)^2 x^2/(3π)^2 x^2/(4π)^2
- x^2/(1π)^2 x^2/(3π)^2 x^2/(4π)^2 - x^2/(1π)^2 x^2/(2π)^2 x^2/(4π)^2
- x^2/(1π)^2 x^2/(2π)^2 x^2/(3π)^2
x^4の項
+ x^2/(6π)^2 x^2/(7π)^2 + x^2/(5π)^2 x^2/(7π)^2 + x^2/(4π)^2 x^2/(7π)^2
+ x^2/(3π)^2 x^2/(7π)^2 + x^2/(2π)^2 x^2/(7π)^2 + x^2/(1π)^2 x^2/(7π)^2
+ x^2/(5π)^2 x^2/(6π)^2 + x^2/(4π)^2 x^2/(6π)^2 + x^2/(3π)^2 x^2/(6π)^2
+ x^2/(2π)^2 x^2/(6π)^2 + x^2/(1π)^2 x^2/(6π)^2 + x^2/(4π)^2 x^2/(5π)^2
+ x^2/(3π)^2 x^2/(5π)^2 + x^2/(2π)^2 x^2/(5π)^2 + x^2/(1π)^2 x^2/(5π)^2
+ x^2/(3π)^2 x^2/(4π)^2 + x^2/(2π)^2 x^2/(4π)^2 + x^2/(1π)^2 x^2/(4π)^2
+ x^2/(2π)^2 x^2/(3π)^2 + x^2/(1π)^2 x^2/(3π)^2 + x^2/(1π)^2 x^2/(2π)^2
x^2の項
- x^2/(7π)^2 - x^2/(6π)^2 - x^2/(5π)^2 - x^2/(4π)^2 - x^2/(3π)^2 - x^2/(2π)^2 - x^2/(1π)^2
定数項
+ 1

x^2(実際はx^3)の項から
(x^2/π^2){1/1^2+1/2^2+1/3^2+1/4^2+1/5^2+1/6^2+1/7^2}=(x^2/π^2) Σ1/n^2はできたすが、
x=4,6(実際はx^5,7)の項からできたせんね。ちなみに、x^4実際はx^5)の項は、オむラヌによるずπ^4/90だそうです。

぀たり、同じ匏から求められたせん。

そこで、オむラヌは、Σ[k=0..∞] 1/(k^4)からx^5の項を求めおいるはずです。

匕甚しお返信線集・削陀(線集枈: 2023幎03月13日 12:26)

はちべえさんぞ。

a_n = (1 +1/n)^n
b_n = (1 +1/n)^(n +1)

ずりいそぎ。

匕甚しお返信線集・削陀(未線集)

Dengan kesaktian Indukmu様、こんにちは。

a_n = (1 +1/n)^n
b_n = (1 +1/n)^(n +1)

で、n→∞ずなるず、a_n=eになりたすね。

a_nのグラフは、http://y-daisan.private.coocan.jp/html/20230313_BMP/2023-3-13-010.png
b_nのグラフは、http://y-daisan.private.coocan.jp/html/20230313_BMP/2023-3-13-011.png
b_n-a_nのグラフは、http://y-daisan.private.coocan.jp/html/20230313_BMP/2023-3-13-012.pngずhttp://y-daisan.private.coocan.jp/html/20230313_BMP/2023-3-13-013.png
です。

b_n-a_n=(1+1/n)^(n+1)-(1+1/n)^n={(1+1/n)-1}(1+1/n)^n=(1/n)(1+1/n)^n
で、n→∞ずなるず、(1/n)(1+1/n)^n=(1/n)eずなっお、無理数ですね。
(1/n)がかかっおいるので、lim(1/n)e→0ですね。

グラフから芋るず、a_nは、割ず早くeに収束したすね。぀たり、無理数に近づくずいうこずですね。

b_n=(1+1/n)a_nですから

a_n<e<b_n  は a_n<e<(1+1/n)a_n 
a_n<e< (1+1/n) a_n=b_n
b_n-a_n は、
0<e-a_n< (1+1/n) a_n-a_n=b_n-a_n
0<e-a_n< (1/n) a_n
さお、nをかけお、
0<n(e-a_n)< a_n
ne-n a_n< a_n
na_nを足しお、
ne< a_n +n a_n
e<(1+1/n)a_n=b_n

なんか行き詰たったなあ。

a_nもb_nもeに近䌌するから、無理数ず蚀えるんじゃないかなあ。

Dengan kesaktian Indukmu様、すみたせん。

匕甚しお返信線集・削陀(線集枈: 2023幎03月13日 15:10)

> x=4,6(実際はx^5,7)の項からできたせんね。

できたすよ。
ずいうか、か぀おここの旧掲瀺板で実際にやったこずがあるので、サむトの方の膚倧な蚘事のどこかに残っおいるはずです。
どれだったかな。

匕甚しお返信線集・削陀(未線集)

できたすよ。

そうなんですか

でも、できるはずがないず思いたす。

匕甚しお返信線集・削陀(線集枈: 2023幎03月14日 07:04)

はちべえさんがおっしゃるに。
a_nもb_nもeに近䌌するから、無理数ず蚀えるんじゃないかなあ。

はちべえさんは
はちべえさんなりの
「閉じおいるこず」に身を捧げなくおはいけないのではないでしょうか

a_nもb_nも有理数に無限回、
四則挔算を適甚したものですから
はちべえさんによる「閉じおいる」定矩によれば
ネむピア数 e もたた有理数であるはずです。

しかるに無理数であるずおっしゃる。
自己矛盟。

原因は、閉じおいるこずに぀いおの
理解䞍足があるのです。

匕甚しお返信線集・削陀(未線集)

Dengan kesaktian Indukmuさた、おはようございたす。

たあ、バヌれル問題は、たた進展がみられたらご報告したす。

匕甚しお返信線集・削陀(線集枈: 2023幎03月14日 20:51)

はちべえさんが考えるずころの
有理数䜓は無限回の四則挔算に぀いお閉じおいる、
に぀いおは、既にネむピア数を䟋に䞊げお
誀りであるずお瀺しさせお頂きたした。

実は、倀のわかっおいる、あるいは倀を蚈算可胜な任意の無理数 c に぀いお
次のこずがいえたす。すなわち。

ある
単調増加有理数列 a_n
単調枛少有理数列 b_n
が存圚しお、任意の正の自然数 n に぀いお
a_n < c < b_n
ずなり、か぀
n → ∞ のずきに
b_n - a_n → 0
ずなる。

a_n も b_n も、無論、c に収束したす。

無理数 c が䞎えられれば、䞊のような、有理数列 a_n や b_n を
高校数孊の範囲でも四則挔算を䜿っお構成可胜なのです。

重ねお匷調しおおきたすが、
はちべえさんが考えるずころの
有理数䜓は無限回の四則挔算に぀いお閉じおいる、ずいう抂念が真ならば
任意の無理数が有理数になっおしたいたす。

バヌれル問題どころの隒ぎではないのです。

無限回の操䜜では、有限回の操䜜たでの感芚が通甚しないこずがたくさんありたす。
このあたりをきちんず教科曞で孊ばないず
人生の貎重な時間が無駄になりたす。

匕甚しお返信線集・削陀(未線集)

はちべえさんにご理解頂きたいこずをもうひず぀。

実数、a、b、ただし、b > a に぀いお以䞋がいえたす。

任意の正数 𝜀 に぀いお
𝑏 - 𝑎 < 𝜀
であるならば
𝑏 - 𝑎 = 0
である。きちんず曞けば

∀𝜀 >0; 𝑏 - 𝑎 < 𝜀 ⇒ 𝑏 - 𝑎 = 0

これを疑っおも益がありたせん。
事実䞊、公理だず思っお䞋さい。

はちべえさんが反䟋を芋出すこずは䞍可胜です。もしもただしい反䟋があれば
䞭間倀の定理やロヌルの定理、平均倀の定理、テヌラヌ展開にた぀わる定理、リヌマン積分にた぀わる定理など、
もろもろ党お、真理倀が疑、になりたす。

最近の、はちべえさんによる数の䜓系に぀いおの
䞀連の疑矩、蚎えは、党お

∀𝜀 >0; 𝑏 - 𝑎 < 𝜀 ⇒ 𝑏 - 𝑎 = 0

ぞの異議申し立おになっおいるのです。

匕甚しお返信線集・削陀(未線集)

倱瀌いたしたした。

実数、a、b、ただし、b > a に぀いお以䞋がいえたす。

ではなく

実数、a、b、ただし、b ≧ a に぀いお以䞋がいえたす。

にしおください

匕甚しお返信線集・削陀(未線集)

オむラヌ積は間違いである。

オむラヌ積はhttps://ja.wikipedia.org/wiki/%E3%82%AA%E3%82%A4%E3%83%A9%E3%83%BC%E7%A9%8Dにありたすが、

(-1/2^2)(1-1/3^2)(1-1/5^2)(1-1/7^2)(1-1/11^2)・・・ζ(2)=1 ---(1)
ずなっおいたす。
これより
{(2^2-1)/2^2}{(3^2-1)/3^2}{(5^2-1)/5^2}{(7^2-1)/7^2}{(11^2-1)/11^2}・・・ζ(2)=1 ---(2)
{(2+1)(2-1)/2^2}{(3+1)(3-1)/3^2}{(5+1)(5-1)/5^2}{(7+1)(7-1)/7^2}{(11+1)(11-1)/11^2}・・・ζ(2)=1 ---(3)
より、
{(2+1)(2-1)}{(3+1)(3-1)}{(5+1)(5-1)}{(7+1)(7-1)}{(11+1)(11-1)}・・・ζ(2)=2^2 3^2 5^2 7^2 11^2・・・・
---(4)
ずなりたす。巊蟺は玠数の前埌の数の積で、{(2+1)(2-1)}を陀いお、すべお偶数ですね。䞀方右蟺はすべおの玠数の2乗の積ですね。

ずころでオむラヌが求めたζ(2)=π^2/6です。この倀はバヌれル問題で求められたした。ζ関数ずは、無関係に求められおいたす。
オむラヌは、バヌれル問題から発展させお、オむラヌ積を芋぀けるのです。
オむラヌ積に感動しおリヌマンはζ関数を進めおゆくのです。
ですから、ζ(2)ずいう衚珟は誀解を招くかもしれたせんね。

ここで、(4)匏の右蟺には、偶数は2^2しかないのに、巊蟺は、巊蟺は玠数の前埌の数の積で、{(2+1)(2-1)}を陀いお、すべお偶数ですね。
それは、2の指数が巊蟺ず右蟺では明らかに違いたすね。

したがっお、この匏は成り立ちたせんね。぀たり、オむラヌ積は間違いであるずいうこずです。

ずいうこずは、バヌれル問題も間違いであり、リヌマンれヌタ関数も間違いであるずいうこずですね。

たあ、無限積であるから、指数が違うず蚀っおも抌し切られおしたうでしょうね、オむラヌ積は問題ないず。

(4)匏は巊蟺が無理数、右蟺が自然数ずいっおも、無意味かな・・・・

虚しい努力か・・・・・

匕甚しお返信線集・削陀(線集枈: 2023幎03月12日 07:33)

> 右蟺が自然数

これを蚌明しおください。
私が知る限りでは、自然数の無限積が自然数になるずいう蚌明は存圚したせんので、今のずころこれははちべえさんが「正しくあっおほしいこず」でしかありたせん。

远加蚌明が必芁な点は他にもあるず思いたすが、最倧の問題点はたずここです。

匕甚しお返信線集・削陀(未線集)

DD++様、おはようございたす。

たず、無限積に぀いお、玠数は玠数定理より数が求められたすが、無限でした。
カントヌルに蚀わせれば、この無限は付加番無限なんでしょうね。぀たり、数えられる皋床の無限ですね。

数孊的垰玍法は、自然数範囲で成り立぀ものなら、自然数はカントヌルの可付番無限であり、数えられるものですね。するず、可付番無限であるから、自然数範囲なので、可付番無限皋床なら、数孊的垰玍法は、䜿えるんじゃないかなず思ったりしたすが、どうなんでしょうね。

䜙談ですが、有理数も、可付番無限でありたす。

匕甚しお返信線集・削陀(線集枈: 2023幎03月12日 09:35)

「数孊的垰玍法はなぜそれで蚌明されたこずになるのか」を考えたこずはあるでしょうか。
たた、「無限ずは䜕であるか」を考えたこずは。

これらに぀いお、䞀般的にどのように考えられおいるかをきちんず理解すれば、加算無限で数孊的垰玍法を䜿えるわけがないずいうこずが玍埗できるず思いたす。
ですからたずはその蟺りを調べおみおはどうでしょう。

匕甚しお返信線集・削陀(未線集)

加算無限は、可算無限ですか

集合論らしいですね。

カントヌルの可付番無限番号を぀けお数えられる無限ずどう違うのですか

集合の芁玠に番号を぀けお、数えられる無限だそうです。

おなじようですね。

匕甚しお返信線集・削陀(線集枈: 2023幎03月12日 09:33)

ああ、倉換ミスしおたしたね。
倱瀌したした、「可算無限」です。

匕甚しお返信線集・削陀(未線集)

オむラヌ積が、認められ、リヌマンのれヌタ関数が認められおいる以䞊、無限の問題はないず蚀えたす。
リヌマンのれヌタ関数でも、各分数は
kは玠数なので、
(k^s-1)k^s=(k-1){k^(n-1)+k^(n-2)+k^n-3)+・・・+k^2+k+1}/k^s
kが2を陀いお、奇数の玠数は(k-1)が偶数なので、本質的に違いはありたせん。

したがっお、リヌマンのれヌタ関数も間違っおいたす。

ζ(s)が、どうであれ、等匏ずしお、巊蟺ず右蟺の2のべき乗数が違うので、成り立ちたせん。

ζ(s)が、2のべき乗であるこずはないので、問題ありたせん。

ずはいえ、たあ、私は、どこかに論文を発衚するわけでもないし、そういう目的もないのですから。

匕甚しお返信線集・削陀(線集枈: 2023幎03月15日 21:33)

今の話題は「数孊的垰玍法は無限でも有効か」ですよね。
オむラヌ積やれヌタ関数のどこで数孊的垰玍法が甚いられおいるのですか

> 私は、どこかに論文を発衚するわけでもないし、そういう目的もないのですから。

論文ずいう圢でなくおも、ここぞの投皿は十分「発衚」に該圓するでしょう。
発衚である以䞊、可胜な限り正しくあろうずする心構えは必芁です。

匕甚しお返信線集・削陀(未線集)

DD++様、おはようございたす。

今の話題は「数孊的垰玍法は無限でも有効か」ですよね。

そうなっおいたすね。私ずしおは、ただ無限ずいう挠然ではなく、たずえば、可付番無限に絞った堎合、オむラヌ積もリヌマンのれヌタ関数も自然数の範囲なのでしょうか、わかりたせんが、無限ずいうこずを気にするこずなく成立しおいるのは事実です。

ですから、無限の皮類に぀いお、怜蚎する必芁があるでしょう。

オむラヌ積やれヌタ関数のどこで数孊的垰玍法が甚いられおいるのですか

぀かわれおいたせん。

論文ずいう圢でなくおも、ここぞの投皿は十分「発衚」に該圓するでしょう。
発衚である以䞊、可胜な限り正しくあろうずする心構えは必芁です。

ご指摘、理解できたした。

匕甚しお返信線集・削陀(線集枈: 2023幎03月16日 07:33)

> 無限ずいうこずを気にするこずなく成立しおいるのは事実です。

違いたす。
無限ずいうこずを気にした䞊で、非垞に慎重に論理を確認した䞊で成立しおいたす。

無限ずいうこずをある皋床軜く扱っおわかりやすく「説明」や「解説」をするこずはありたすが、それらは「蚌明」ではないのです。

匕甚しお返信線集・削陀(線集枈: 2023幎03月16日 22:34)

DD++様、おはようございたす。

無限ずいうこずを気にした䞊で、非垞に慎重に論理を確認した䞊で成立しおいたす。

それは、オむラヌもリヌマンもどのようにしお、裏付けられたのですか

無限ずいうこずをある皋床軜く扱っおわかりやすく「説明」や「解説」をするこずはありたすが、それらは「蚌明」ではないのです。

確かに、改めおそれを蚌明されおいたせんね。

匕甚しお返信線集・削陀(未線集)

> オむラヌもリヌマンもどのようにしお、裏付けられたのですか

掲瀺板のレス皋床に収たる話じゃないので、「無限数列の絶察収束」「耇玠関数の解析接続」にきちんず觊れながら蚌明しおいる曞籍等を探しおみおください。

> 改めおそれを蚌明されおいたせんね。

蚌明はされおいたす。
はちべえさんが読んだこずないだけです。

匕甚しお返信線集・削陀(未線集)

数の切断

空欄がn個あり
□□□□
䞭に1nの数字が぀ず぀党お入る。
このずき出来るn桁の敎数で、以䞋の条件がすべお成り立぀のもを探しお䞋さい。

条件a䞊2桁が2で割れる。
     侊3桁が3で割れる。
     侊4桁が4で割れる。
     
     n桁がnで割れる。

(1)n=3
(2)n=4
(3)n=5
(4)n=6
(5)n=7
(6)n=8
(7)n=9
での敎数はそれぞれ䜕


次にこの条件を

条件b䞋2桁が2で割れる。
     例3桁が3で割れる。
     例4桁が4で割れる。
     
     n桁がnで割れる。

ずし
n=9で,
この条件を党お満たす9桁の敎数の䞭で探そうずするず
例2桁(最埌が偶数ず䞋5桁(最埌が5)ずなり盞反する。
そこで䞋5桁の条件は陀倖し、その他は条件が満たされる9桁の敎数の䞭で
最小ず最倧のものは䜕でしょう

匕甚しお返信線集・削陀(線集枈: 2023幎03月15日 09:52)

昔、どこかで聞いたような問題ですね
(1) n=3 のずき、 桁の数は、の倍数なので、各䜍の数の和は、の倍数
 可胜性は、 、、、、、、、、 の䜕れか。
 䞊2桁が2で割れるので、䞊2桁目は、 、、、、 の䜕れか。
 䜿える数字は、で、各䜍の数が党お盞異なるこずに泚意しお、
 䞊2桁目が  のずき、桁の数の可胜性は、
 、
 以䞊から、桁の敎数の䞭で、最小倀は、 で、最倧倀は、 である。
(2) n=4 のずき、䜿える数字は、、、、で、各䜍の数が党お盞異なり、
侊2桁が2で割れる。
侊3桁が3で割れる。
4桁が4で割れる。
ずいう条件から、たず、4桁が4で割れるためには、䞋2桁が4の倍数であればよいので、
その可胜性は、 ○○、○○、○○ の通り
○○が、の倍数ずなるこずはない。
○○が、の倍数ずなるのは、 、
○○が、の倍数ずなるこずはない。
 以䞊から、 、 で、この䞭に、䞊2桁が2で割れるものはない。
 よっお、n=4 のずき、解なし
(3) n=5 のずき、䜿える数字は、、、、、で、各䜍の数が党お盞異なり、
侊2桁が2で割れる。
侊3桁が3で割れる。
侊4桁が4で割れる。
5桁が5で割れる。
ずいう条件から、䞀の䜍は5ず確定し、n=4 のずきず同様に、解なしずなる。
(4) n=6 のずき、䜿える数字は、、、、、、で、各䜍の数が党お盞異なり、
侊2桁が2で割れる。
侊3桁が3で割れる。
侊4桁が4で割れる。
侊5桁が5で割れる。
6桁が6で割れる。
ずいう条件から、十の䜍は5ず確定し、千の䜍癟の䜍も可胜性は、
 、、、、、
 たた、䞇の䜍の可胜性は、、、 なので、以䞊を組み合わせるず、可胜性は、
 ○○、○○、○○、○○、○○、○○、
 ○○、○○、○○、○○
 残りの数字を曞き加えお、条件を満たすものを探すず、
  、
の通り存圚する。
(5) n=7 のずき、䜿える数字は、、、、、、、で、各䜍の数が党お盞異なり、
侊2桁が2で割れる。
侊3桁が3で割れる。
侊4桁が4で割れる。
侊5桁が5で割れる。
侊6桁が6で割れる。
7桁が7で割れる。
ずいう条件から、十の䜍は5ず確定し、n=6 のずきず同様に考えお、可胜性は、
 ○○○、○○○、○○○、○○○、○○○、○○○、
 ○○○、○○○、○○○、○○○、○○○、○○○、
 ○○○、○○○
 残りの数字を曞き加えお、条件を満たすものはないので、解なしずなる。
(6) n=8 のずき、䜿える数字は、、、、、、、、で、各䜍の数が党お盞異なり、
侊2桁が2で割れる。
侊3桁が3で割れる。
侊4桁が4で割れる。
侊5桁が5で割れる。
侊6桁が6で割れる。
侊7桁が7で割れる。
8桁が8で割れる。
ずいう条件から、十の䜍は5ず確定し、n=7 のずきず同様に考えお、可胜性は、
 ○○○○、○○○○、○○○○、○○○○、○○○○、
 ○○○○、○○○○、○○○○、○○○○、○○○○、
 ○○○○、○○○○、○○○○、○○○○、○○○○、
 ○○○○、○○○○、○○○○、○○○○、○○○○、
 ○○○○、○○○○、○○○○、○○○○、○○○○、
 ○○○○、○○○○、○○○○、○○○○、○○○○
 残りの数字を曞き加えお、条件を満たすものを探すず、  の通り存圚する。

匕甚しお返信線集・削陀(線集枈: 2023幎03月16日 11:02)

条件aに぀いお
(1)(7)共通
条件から、偶数桁目は偶数でなければならない。 (a)
条件から、䞊から3桁ごずの合蚈は3の倍数でなければならない。 (b)
(3)(7)共通
条件から、5桁目は5でなければならない。 (c)

(1)
(a)から123,321しかあり埗ないが、これはどちらも条件を満たす。

(2)
(a)から4桁目は2か4だが、䞊3桁が3で割り切れるためには4桁目は4でなければならない。
しかし3桁目が奇数で4桁目が4である数は4で割り切れないので解なし。

(3)
(c)から5桁目は5なので䞊4桁は(2)を満たさなければならない。よっお解なし。

(4)
䜿える偶数は2,4,6なので、(a)(b)(c)から、䞋3桁は456たたは654でなけれぱならない。
3桁目が奇数で4桁目が4だず䞊4桁が4で割り切れないので、䞋3桁は654ず決たる。
するず䞊3桁は1,2,3ずなり(1)を満たす必芁があるので、䞊3桁は123か321。
埓っお条件を満たす解は123654ず321654。

(5)
䜿える偶数は(4)ず同じなので、4桁目6桁目は654。
残る1,2,3,7は足しお3で割るず1䜙るので、7桁目は1か7。
(a)により、7桁目が1の堎合は先頭3桁は327か723、7桁目が7の堎合は先頭3桁は123か321ず決たるが、
3276541,7236541,1236547,3216547はいずれも7で割り切れず、解なし。

(6)
䜿える偶数は2,4,6,8であり、3桁目が奇数で䞊4桁が4で割り切れなければならないこずから
4桁目は2か6なので、(a)(b)(c)から4桁目6桁目は258か654。
258のずき残る数は1,3,4,6,7だが、7桁目が奇数で䞊8桁が8で割り切れるためには、
少なくずも8桁目は4で割り切れない偶数すなわち6でなければならず、
(a)(b)から14725836,74125836
654のずき残る数は1,2,3,7,8だが、7桁目が奇数で䞊8桁が8で割り切れるためには、
少なくずも8桁目は4で割り切れない偶数すなわち2でなければならず、
(a)(b)から18365472,38165472,38765412,78365412
このうち「䞊7桁が7で割り切れ䞊8桁が8で割り切れる」を満たすものは38165472のみ。

(7)
(6)ず同様に xxx258xxx, xxx654xxx
7桁目が奇数で䞊8桁が8で割り切れるためには xxx258x6x, xxx654x2x
残りの偶数を2桁目に入れお x4x258x6x, x8x654x2x
前者は1桁目ず3桁目に1,7を入れなければならないが、
(6)から䞊8桁が14725836,74125836は条件を満たさないので、可胜性があるのは
147258963,741258963
しかしいずれも「䞊7桁が7で割り切れる」ずいう条件を満たさない。
埌者は1桁目ず3桁目に(1たたは7)(3たたは9)を入れなければならないので、可胜性があるのは
183654729,183654927,189654327,189654723,381654729,381654927,387654129,387654921,
783654129,783654921,789654123,789654321,981654327,981654723,987654123,987654321
このうち䞊7桁が7で割り切れるものは381654729ず783654921だが、埌者は䞊8桁が8で割り切れないので
条件を満たす解は381654729のみ。

(条件aのたずめ)
(1) 123,321
(4) 123654,321654
(6) 38165472
(7) 381654729
(2)(3)(5)は解なし。

匕甚しお返信線集・削陀(未線集)

条件bに぀いお
「䞋2桁が2で割れる」ず「䞋9桁が9で割れる」は無芖しおよい。
「䞋4桁が4で割れる」から
xx12,xx32,xx52,xx72,xx24,xx64,xx84,xx16,xx36,xx56,xx76,xx28,xx48,xx68
「䞋3桁が3で割れる」の条件を加えお
x312,x612,x912,x132,x432,x732,x852,x372,x672,x972,x324,x624,
x924,x264,x564,x864,x384,x684,x984,x216,x516,x816,x936,x156,
x456,x756,x276,x576,x876,x528,x348,x648,x948,x168,x468,x768
このうち「䞋8桁が8で割れる」ものはちょうど半数で
x312,x912,x432,x672,x624,x264,x864,x384,x984,x216,x816,x936,
x456,x576,x528,x648,x168,x768
残る条件は「䞋6桁が6で割れる」「䞋7桁が7で割れる」だが、
手䜜業で党通り蚈算するのは非垞に倧倉なので最小倀ず最倧倀だけを考えるこずにする。
「䞋6桁が6で割れる」ずいう条件から䞊3桁の合蚈は3の倍数でなければならないので、䞊3桁は
小さい順に123,126,129,132,135,138,

倧きい順に987,984,981,978,975,972,

侊3桁が123の堎合、䞋3桁は䞊蚘のうち1,2,3を含たないものなので
864,984,456,576,648,768
䞊から4桁目が4だずするず䞋3桁は576か768なので
123489576,123498576,123459768,123495768
しかしこれらはいずれも䞋7桁が7で割り切れない。
䞊から4桁目が5だずするず䞋3桁は864,984,648,768なので候補は
123579864,123597864,123567984,123576984,123579648,123597648,123549768,123594768
このうち123567984だけ䞋7桁が7で割り切れお条件bを満たすので、これが最小倀。
侊3桁が987の堎合、䞋3桁は䞊蚘のうち9,8,7を含たないものなので
312,432,624,264,216,456
䞊から4桁目が6だずするず、䞋3桁は312か432なので
987654312,987645312,987651432,987615432
しかしこれらはいずれも䞋7桁が7で割り切れない。
䞊から4桁目が5だずするず、䞋3桁は312,432,624,264,216なので候補は
987564312,987546312,987561432,987516432,987531624,
987513624,987531264,987513264,987543216,987534216
このうち䞋7桁が7で割り切れるものは987564312ず987516432の二぀であり、
987564312の方が倧きいのでこれが最倧倀。

(条件bの答え)
最小倀は 123567984 、最倧倀は 987564312

匕甚しお返信線集・削陀(未線集)

πの出珟

ζ(3)=1 + 1/2^3 + 1/3^3 + 1/4^3 + 1/5^3 + 1/6^3 + 1/7^3 +
にはπの姿は珟れないが
1 - 1/3^3 + 1/5^3 - 1/7^3 + 1/9^3 -=π^3/32
には、ちゃんずπが出珟しおくる。

ここに
S1=1/(1^3*2^3) + 1/(2^3*3^3) + 1/(3^3*4^3) + 1/(4^3*5^3) +
S2=1/(1^3*3^3) + 1/(2^3*4^3) + 1/(3^3*5^3) + 1/(4^3*6^3) +

にもπは姿を珟したす。
ではどんなものになるでしょうか

匕甚しお返信線集・削陀(未線集)

S1 = 10 - π^2 、S2 = (21 - 2π^2)/32 ですか

匕甚しお返信線集・削陀(線集枈: 2023幎03月15日 01:59)

1/(n^3(n+1)^3)=(6n^2-3n+1)/n^3-(6(n+1)^2+3(n+1)+1)/(n+1)^3
なので
S1=Σ[n=1∞]1/(n^3(n+1)^3)
=Σ[n=1∞](6n^2-3n+1)/n^3-(6(n+1)^2+3(n+1)+1)/(n+1)^3
=(6*1^2+3*1+1)/1^3-6ζ(2)
=10-π^2

1/(n^3(n+2)^3)=(1/16){(3n^2-3n+2)/n^3-(3(n+2)^2+3(n+2)+2)/(n+2)^3}
なので
S2=Σ[n=1∞]1/(n^3(n+2)^3)
=(1/16)Σ[n=1∞]{(3n^2-3n+2)/n^3-(3(n+2)^2+3(n+2)+2)/(n+2)^3}
=(1/16){(3*1^2+3*1+2)/1^3+(3*2^2+3*2+2)/2^3-6ζ(2)}
=(21-2π^2)/32

匕甚しお返信線集・削陀(線集枈: 2023幎03月14日 23:07)
合蚈1736件 (投皿283, 返信1453)

ロケットBBS

Page Top